Difference between revisions of "2006 AMC 12A Problems/Problem 8"

(Solution 3)
(Solution 3)
Line 49: Line 49:
 
Since there are 3 possibilities the answer is <math>\boxed{\textbf{(C) }3}.</math>
 
Since there are 3 possibilities the answer is <math>\boxed{\textbf{(C) }3}.</math>
 
==Solution 3==
 
==Solution 3==
!!! I might have a solution but am not sure if this is a correct way to solve it. !!!
 
 
 
We want some consecutive numbers that add up. We can use casework to solve this. We know that any set of 6 consecutive, positive integers cannot be greater than 5. Therefore the max amount of elements is 5.
 
We want some consecutive numbers that add up. We can use casework to solve this. We know that any set of 6 consecutive, positive integers cannot be greater than 5. Therefore the max amount of elements is 5.
  

Revision as of 23:29, 17 December 2024

The following problem is from both the 2006 AMC 12A #8 and 2008 AMC 10A #9, so both problems redirect to this page.

Problem

How many sets of two or more consecutive positive integers have a sum of $15$?

$\textbf{(A) } 1\qquad \textbf{(B) } 2\qquad \textbf{(C) } 3\qquad \textbf{(D) } 4\qquad \textbf{(E) }  5$

Solution 1

Notice that if the consecutive positive integers have a sum of $15$, then their average (which could be a fraction) must be a divisor of $15$. If the number of integers in the list is odd, then the average must be either $1, 3,$ or $5$, and $1$ is clearly not possible. The other two possibilities both work:

  • $1 + 2 + 3 + 4 + 5 = 15$
  • $4 + 5 + 6 = 15$

If the number of integers in the list is even, then the average will have a $\frac{1}{2}$. The only possibility is $\frac{15}{2}$, from which we get:

  • $15 = 7 + 8$

Thus, the correct answer is $\boxed{\textbf{(C) }3}.$


Question: (RealityWrites) Is it possible that the answer is $4$, because $0+1+2+3+4+5$ should technically count, right?

Answer: (IMGROOT2) It isn't possible because the question asks for positive integers, and this means that negative integers or zero aren't allowed.

Note to readers: make sure to always read the problem VERY carefully before attempting; it could mean the difference of making the cutoff.

Solution 2

Any set will form a arithmetic progression with the first term say $a$. Since the numbers are consecutive the common difference $d = 1$.

The sum of the AP has to be 15. So,

\[S_n = \frac{n}{2} \cdot (2a + (n-1)d)\] \[S_n = \frac{n}{2} \cdot (2a + (n-1)1)\] \[15 = \frac{n}{2} \cdot (2a + n - 1)\] \[2an + n^2 - n = 30\] \[n^2 + n(2a - 1) - 30 = 0\]

Now we need to find the number of possible sets of values of a, n which satisfy this equation. Now $a$ cannot be 15 as we need 2 terms. So a can only be less the 15.

Trying all the values of a from 1 to 14 we observe that $a = 4$, $a = 7$ and $a = 1$ provide the only real solutions to the above equation.The three possibilites of a and n are.

\[(a,n) = (4,3),(7, 2),(1, 6)\]

The above values are obtained by solving the following equations obtained by substituting the above mentioned values of a into $n^2 + n(2a - 1) - 30 = 0$,

\[n^2 + 7n - 30 = 0\] \[n^2 + 13n - 30 = 0\] \[n^2 - n - 30 = 0\]

Since there are 3 possibilities the answer is $\boxed{\textbf{(C) }3}.$

Solution 3

We want some consecutive numbers that add up. We can use casework to solve this. We know that any set of 6 consecutive, positive integers cannot be greater than 5. Therefore the max amount of elements is 5.

Case 1: There are 5 numbers

-We know that 15/5 = 3, so if we put 3 as our middle number and expand evenly, we get {1,2,3,4,5}

Case 2: There are 4 numbers

-By a bit of bashing we find that there are no solutions

Case 3: There are 3 numbers

-We know that 15/3 = 5, so if we put 5 as our middle number and expand evenly, we get {4,5,6}

Case 4: There are 2 numbers

-We'll say x is the lower number. Thus x + (x + 1) = 15. Solving for x, x = 7. Thus getting {7,8}

Adding up all the cases we get 1 + 0 + 1 + 1 = 3

See also

2006 AMC 12A (ProblemsAnswer KeyResources)
Preceded by
Problem 7
Followed by
Problem 9
1 2 3 4 5 6 7 8 9 10 11 12 13 14 15 16 17 18 19 20 21 22 23 24 25
All AMC 12 Problems and Solutions
2006 AMC 10A (ProblemsAnswer KeyResources)
Preceded by
Problem 8
Followed by
Problem 10
1 2 3 4 5 6 7 8 9 10 11 12 13 14 15 16 17 18 19 20 21 22 23 24 25
All AMC 10 Problems and Solutions

The problems on this page are copyrighted by the Mathematical Association of America's American Mathematics Competitions. AMC logo.png